LSAT and Law School Admissions Forum

Get expert LSAT preparation and law school admissions advice from PowerScore Test Preparation.

 Administrator
PowerScore Staff
  • PowerScore Staff
  • Posts: 8919
  • Joined: Feb 02, 2011
|
#41405
Please post your questions below!
 allicr
  • Posts: 5
  • Joined: Oct 14, 2018
|
#60067
I diagrammed this one as follows, but I cannot figure out how to combine statements to make A into a CBT.

1. Bill --> (most) prepared to vote
2. Bill not likely get approval --> compromise possible --> law
3. Bill has issues --> compromise impossible


Also, any tips for solving a question like this under timing pressure?

Thank you!
 Rachael Wilkenfeld
PowerScore Staff
  • PowerScore Staff
  • Posts: 1358
  • Joined: Dec 15, 2011
|
#60205
Hi allicr,

I think your diagramming shows a strong understanding of conditional reasoning. Mine looks slightly different, but has the same essential features.

When I was thinking about a prephrase here, I immediately noticed that two of the statements had conditions related to compromise. In the first, it says that when a bill is at first unlikely to be approved, compromise is usually possible, allowing the bill to become a law. The second conditional says that if the bill concerns an issue of fundamental importance to a large bloc, compromise is impossible. My prephrase was essentially noting that potential contradiction. In one, the result that compromise is possible. The second, it's that compromise is impossible.

I looked through the answer choices with that focus in mind, and looked first at the ones that included the ideas of compromise. That left me with answer choices (A) and (C) as starting points. Answer choice (A) says compromises regarding issues of fundamental importance usually don't enable the law to pass. We don't know if that can be true or not, because the passage says those compromises don't occur. So we can't say what would happen if they DID occur.

Answer choice (C) however, states something that we know cannot occur. It says that most bills containing an issue of fundamental importance pass as a result of compromise. But we know there can't be compromise. If the bill contains an issue of fundamental importance, compromise is impossible. So this is the correct answer.

In terms of speed, I think a lot can be gained from prephrasing these. I read through all 5 answer choices, but I focus first on thinking about those that are most likely to be correct.

Hope that helps!
Rachael
 allicr
  • Posts: 5
  • Joined: Oct 14, 2018
|
#60212
Thank you, Rachael! That was incredibly helpful.
 g_lawyered
  • Posts: 211
  • Joined: Sep 14, 2020
|
#93631
Hi P.S.,
I'm a stuck on why answer choice A is a Could Be True and an incorrect answer to this Must Be False question. Specifically, I didn't understand this part of the explanation:
Answer choice (A) says compromises regarding issues of fundamental importance usually don't enable the law to pass. We don't know if that can be true or not, because the passage says those compromises don't occur. So we can't say what would happen if they DID occur.
.

How can we connect the conditional statements to form a logical chain to make A something that is possible?

Here's how I broke it down:
Premise 2: Bill not likely to approve :arrow: compromise possible :arrow: pass into law
Contrapositive of premise 2: NOT Pass into law :arrow: Compromise impossible :arrow: bill likely to approve

Premise 3: Bill important to large group of rep. :arrow: Compromise impossible
Contrapositive of premise 3: Compromise possible :arrow: Bill NOT important to large group of rep.

With these conditional statements how can we infer what answer choice A is saying can be true? :-?

Also, because this question asks for a statement that must be false, Why do we eliminate answer choice D and E? I eliminated both answer choices because I thought they were irrelevant/ couldn't be disproven based on the statements. Is this correct reason to eliminate these answer choices? If answer choice doesn't state a could be true statement, does that mean it's a could be false? I hope my reasoning makes sense. :-?
Any clarification would help.

Thanks in advance!
 Adam Tyson
PowerScore Staff
  • PowerScore Staff
  • Posts: 5153
  • Joined: Apr 14, 2011
|
#93847
An answer that is irrelevant or cannot be disproven is therefore something that could be true, GGIBA003@FIU.EDU. Any answer that is not directly in conflict with the facts could be true. Answer A could be true not because some conditional relationship shows that it is possible, but because none of the conditional relationship prove that it is impossible.

We cannot prove a Mistaken Reversal, but it could be true.

We cannot prove a Mistake Negation, but it could be true.

We cannot prove something that is not discussed in the stimulus, but it could be true (because we also cannot disprove it).

Answer A could be true because it's talking about something a little different from what the stimulus is talking about. The only thing the stimulus tells us about fundamental issues is that content of bills concerning those issues cannot be compromised, It tells us nothing about how compromises on those issues may or may not affect the passage of any other bills. The stimulus never proves, or disproves, answer A, and that's why it could be true.

Imagine an answer that said "Finland is a great place for a winter vacation." That could be true, because the stimulus doesn't prove it isn't true, and that would therefore be a wrong answer to this Must Be False question.
 g_lawyered
  • Posts: 211
  • Joined: Sep 14, 2020
|
#93858
This is something I haven't learned about Adam! So it makes sense why I thought irrelevant answer choices were wrong (not could be true answer). I now understand why answer choice A is a could be true? But if answer choice D and E aren't disproven? Why are these answer choices incorrect? I noticed both answer choices test out the last sentence of the stimulus...

Thanks in advance!
User avatar
 Beth Hayden
PowerScore Staff
  • PowerScore Staff
  • Posts: 123
  • Joined: Sep 04, 2021
|
#93879
Hi GGIBA,

An answer choice is wrong on a must be false question if there is any chance at all that it could be true. As Adam said, that means if you can't disprove it, it's not the right answer. If the answer choice talks about something that isn't really addressed in the stimulus, we have no way of proving or disproving it. That is exactly what is going on with D and E.

We don't know enough in the stimulus to prove or disprove D. The stimulus tells us something about bills concerning issues of fundamental importance, but it doesn't tell us how many bills fall in that category. Maybe that's 90% of bills, maybe it's 5% of bills, we really have no idea. E is essentially the negation of D, so it has the exact same problem.

Hope that helps!
Beth
 g_lawyered
  • Posts: 211
  • Joined: Sep 14, 2020
|
#93905
I noticed that D and E are stating the opposite of each other. However, it sounds like the reason answer choice A is the correct answer could be applied to answer choice D & E (it's not proven nor disproven). It mentions something different than what the argument states. So, why is answer choice A better than D or E?
Thanks in advance!
 Adam Tyson
PowerScore Staff
  • PowerScore Staff
  • Posts: 5153
  • Joined: Apr 14, 2011
|
#93917
A is not better than D or E, and not worse, because all three answers could be true and are therefore wrong answers. All the wrong answers are equally wrong! The only one that's better is the right one, which is the one that must be false - answer C.

Get the most out of your LSAT Prep Plus subscription.

Analyze and track your performance with our Testing and Analytics Package.